0 Daumen
308 Aufrufe

Zeigen Sei, dass für alle n ∈ ℕ \ {0} die Ungleichungen

(a) \( \sum\limits_{k=1}^{n}{\frac{1}{k^2}} \) ≤ 2 - \( \frac{1}{n} \)   und

(b) \(\frac{3n}{2n+1}\) ≤ \( \sum\limits_{k=1}^{n}{\frac{1}{k^2}} \)

erfüllt sind.


Danke im Voraus



Avatar von

3 Antworten

0 Daumen

Hallo Markus,

Du hast ja einige dieser Aufgaben heute ... das ist etwas schwieriger. Zu zeigen ist$$\sum\limits_{k=1}^{n}{\frac{1}{k^2}} \stackrel{?}{\le} 2 - \frac{1}{n}, \quad n \in \mathbb N$$Man zeigt zunächst wieder, dass die Gleichung für \(n=1\) aufgeht$$\sum\limits_{k=1}^{1}{\frac{1}{k^2}} \le 2 - \frac{1}{1} = 1\space \checkmark$$Im folgenden zeigt man nun, dass das auch ok ist, wenn man \(n\) durch \(n+1\) ersetzt. Also es soll sein$$\sum\limits_{k=1}^{n+1}{\frac{1}{k^2}} \stackrel{?}{\le} 2 - \frac{1}{n+1}$$Dafür darf man davon ausgehen, dass es für \(\sum\limits_{k=1}^{n}{\frac{1}{k^2}} \le 2 - \frac{1}{n}\) bereits bewiesen ist.$$\begin{aligned} \sum\limits_{k=1}^{n+1}{\frac{1}{k^2}} &= \sum\limits_{k=1}^{n}{\frac{1}{k^2}} \space +\frac 1{(n+1)^2} \\ &\le 2 - \frac{1}{n}  +\frac 1{(n+1)^2} \\ &= 2 \underbrace{- \frac 1{n+1} + \frac 1{n+1}}_{=0} - \frac{1}{n}  +\frac 1{(n+1)^2} \\ &= 2 - \frac 1{n+1} + \frac{n(n+1)}{n(n+1)^2} - \frac{(n+1)^2}{n(n+1)^2} + \frac n{n(n+1)^2}\\ &= 2 - \frac 1{n+1} +\frac{n^2+n - n^2-2n-1 + n}{n(n+1)^2}\\ &= 2 - \frac 1{n+1} +\frac{-1}{n(n+1)^2}\\ &\le 2 - \frac 1{n+1}\\ &\text{q.e.d.} \end{aligned}$$Versuche mal die Aufgabe (b) im gleichem Stil zu bearbeiten.


Nachtrag:

Wobei das doch etwas schwieriger ist. Induktionsanfang sollte klar sein. Ich zeige hier nur den Übergang von \(n\) nach \(n+1\). Es soll gezeigt werden, dass $$\frac{3(n+1)}{2(n+1)+1}\le \sum\limits_{k=1}^{n+1}{\frac{1}{k^2}}$$Es wäre wahrscheinlich einfacher mit der Summe zu beginnen, aber so geht es auch; ich beginne mit dem Bruch:$$\begin{aligned} \frac{3(n+1)}{2(n+1)+1} &= \frac{3(n+1)}{2n+3}\\ &= \underbrace{\frac{3n}{2n+1} - \frac{3n}{2n+1}}_{=0} + \frac{3(n+1)}{2n+3} \\ &\le \sum\limits_{k=1}^{n}{\frac{1}{k^2}} - \frac{3n}{2n+1} + \frac{3(n+1)}{2n+3} \\ &= \sum\limits_{k=1}^{n}{\frac{1}{k^2}} + \underbrace{\frac 1{(n+1)^2} - \frac 1{(n+1)^2}}_{=0} - \frac{3n}{2n+1} + \frac{3(n+1)}{2n+3}\\ &=  \sum\limits_{k=1}^{n}{\frac{1}{k^2}} + \frac 1{(n+1)^2} - \frac{(2n+1)(2n+3) +\overbrace{3n(n+1)^2(2n+3) - 3(n+1)}^{3(n+1)\cdot a}}{(n+1)^2(2n+1)(2n+3)}\\ &= \sum\limits_{k=1}^{n+1}{\frac{1}{k^2}} - \frac{(2n+1)(2n+3) + 3(n+1)\overbrace{(n(n+1)(2n+3) -1)}^{\gt 0}}{(n+1)^2(2n+1)(2n+3)}\\ &\lt \sum\limits_{k=1}^{n+1}{\frac{1}{k^2}}\\&\text{q.e.d.} \end{aligned}$$In der 5.Zeile habe ich den Term \(3(n+1)\) aus den letzten beiden Summanden ausgeklammert. Und in der darauf folgenden Zeile ist der letzte Faktor des letzten Summanden sicher \(\gt 0\). Daher brauche ich den Term nicht weiter umzuformen. Da er nur aus Summen und Produkten von positiven Ausdrücke besteht, muss der ganze Bruch \(\gt 0\) sein. Und es gilt \(\sum_{k=1}^{n+1} \frac 1{k^2} + \epsilon \gt \sum_{k=1}^{n+1} \frac 1{k^2}\) wenn \(\epsilon \gt 0\).

Und wenn trotzdem noch was offen ist, so stelle möglichst konkrete Fragen.

Gruß Werner

Avatar von 48 k

Antwort um einen Nachtrag erweitert.

0 Daumen

(a) Beweis per Induktion über \(n\).

Induktionsanfang \(n=1\): \(\sum\limits_{k=1}^{1}\frac{1}{k^2} = 1 \leq 1 = 2-\frac{1}{1}\), damit gilt der IA.

Induktionshypothese: Für ein \(n\in \mathbb{N}\) gilt \(\sum\limits_{k=1}^{1}\frac{1}{k^2}\leq 2-\frac{1}{n}\).

Induktionsschritte \(n\rightarrow n+1\): \(\sum\limits_{k=1}^{n+1} \frac{1}{k^2} = \frac{1}{(n+1)^2} + \sum\limits_{k=1}^{n} \frac{1}{k^2} \overset{IH}{\leq} \frac{1}{(n+1)^2} + (2-\frac{1}{n}) \).

Nun gilt allerdings \(\frac{1}{(n+1)^2} = \frac{1}{n^2+2n+1} \leq \frac{1}{n^2+n} = \frac{1}{n(n+1)} = \frac{n+1-n}{n(n+1)} = \frac{n+1}{n(n+1)} - \frac{n}{n(n+1)} = \frac{1}{n}-\frac{1}{n+1}\).

Damit ergibt sich \(\frac{1}{(n+1)^2} + (2-\frac{1}{n}) \leq \frac{1}{n}-\frac{1}{n+1} + 2 - \frac{1}{n} = 2-\frac{1}{n+1}\), woraus insgesamt \(\sum\limits_{k=1}^{n+1} \frac{1}{k^2} \leq 2-\frac{1}{n+1}\) und entsprechend die Induktionsschritte folgen.


Die Teilaufgabe (b) dann im selben Stil zu (a) (eine weitere Induktion).

Avatar von 2,9 k
0 Daumen

Aloha :)

zu a) Für \(n=1\) rechnet man die Gültigkeit schnell nach. Für \(n\ge2\) betrachte:$$\phantom{=}\sum\limits_{k=1}^n\frac{1}{k^2}=\frac{1}{1^2}+\sum\limits_{k=2}^n\frac{1}{k^2}<1+\sum\limits_{k=2}^n\frac{1}{(k-1)k}=1+\sum\limits_{k=2}^n\left(\frac{1}{k-1}-\frac1k\right)$$$$=1+\sum\limits_{k=2}^n\frac{1}{k-1}-\sum\limits_{k=2}^n\frac1k=1+\sum\limits_{k=1}^{n-1}\frac{1}{k}-\sum\limits_{k=2}^n\frac1k$$$$=1+\left(\frac11+\sum\limits_{k=2}^{n-1}\frac{1}{k}\right)-\left(\sum\limits_{k=2}^{n-1}\frac1k+\frac1n\right)=1+\frac11-\frac1n=2-\frac1n$$

zu b) Hier musst du eigentlich gar nicht viel tun, weil$$\frac{3n}{2n+1}=\frac{3n+\frac32-\frac32}{2n+1}=\frac{\frac32(2n+1)-\frac32}{2n+1}=\frac32-\frac{\frac32}{2n+1}=\frac32-\frac{3}{4n+2}<\frac32$$Das heißt, sobald die Summe den Wert \(\frac32\) überschritten hat, ist die Aussage klar. Bis dahin kannst du die Werte einfach auflisten:$$\begin{array}{r|r|r}n & \frac{3n}{2n+1} & \sum\limits_{k=1}^n\frac1{k^2}\\\hline 1 & 1,0000 & 1,0000\\2 & 1,2000 & 1,2500\\3 & 1,2857 & 1,3611\\4 & 1,3333 & 1,4236\\5 & 1,3636 & 1,4636\\6 & 1,3846 & 1,4914\\\hline7 & 1,4000 & 1,5118\end{array}$$

Avatar von 148 k 🚀

Ein anderes Problem?

Stell deine Frage

Willkommen bei der Mathelounge! Stell deine Frage einfach und kostenlos

x
Made by a lovely community